Find the arc length parametrization of a curve

Click For Summary
The discussion focuses on finding the arc length parametrization of the curve defined by r = (3t cos(t), 3t sin(t), 2√2 t^(3/2)). The integral of the magnitude of the velocity vector simplifies to s(t) = 3t + (3/2)t^2. The challenge arises in solving for t in terms of s, which the original poster finds difficult. Another participant suggests that the issue can be resolved since it is a quadratic equation. The conversation emphasizes the importance of correctly manipulating the equation to find the desired parametrization.
AramN
Messages
1
Reaction score
0

Homework Statement


Find the arc length parametrization of the curve r = (3t cost, 3tsint, 2sqrt(2)t^(3/2) ) .

Homework Equations


s(t)=integral of |r'(t)| dt

The Attempt at a Solution


I was able to get the integral of the magnitude of the velocity vector to simplify to:
s(t) = integral of sqrt(9(1+t)^2) dt
evaluating the integral, s(t) = 3t+(3/2)t^2
In other problems similar to this one I would solve for t in terms of s then put it back into the original equation, but for this one I was unable to solve for t in terms of s.
 
Physics news on Phys.org
AramN said:

Homework Statement


Find the arc length parametrization of the curve r = (3t cost, 3tsint, 2sqrt(2)t^(3/2) ) .

Homework Equations


s(t)=integral of |r'(t)| dt

The Attempt at a Solution


I was able to get the integral of the magnitude of the velocity vector to simplify to:
s(t) = integral of sqrt(9(1+t)^2) dt
evaluating the integral, s(t) = 3t+(3/2)t^2
In other problems similar to this one I would solve for t in terms of s then put it back into the original equation, but for this one I was unable to solve for t in terms of s.

I don't see the problem in solving for t. It's just a quadratic equation.
 
Question: A clock's minute hand has length 4 and its hour hand has length 3. What is the distance between the tips at the moment when it is increasing most rapidly?(Putnam Exam Question) Answer: Making assumption that both the hands moves at constant angular velocities, the answer is ## \sqrt{7} .## But don't you think this assumption is somewhat doubtful and wrong?

Similar threads

Replies
2
Views
2K
  • · Replies 2 ·
Replies
2
Views
2K
  • · Replies 7 ·
Replies
7
Views
1K
Replies
1
Views
1K
  • · Replies 6 ·
Replies
6
Views
2K
  • · Replies 1 ·
Replies
1
Views
2K
  • · Replies 1 ·
Replies
1
Views
2K
Replies
8
Views
2K
Replies
3
Views
2K
  • · Replies 6 ·
Replies
6
Views
2K